Where does a Laurent Series converge?












1












$begingroup$


From the Cauchy integral formula, I get



$f(z)=int_{C} frac{f(zeta)}{zeta -z}dzeta$



manipulating, =$int_{C} f(zeta)(frac{1}{zeta -p -(z-p)})$=$int_{C} f(zeta)(frac{1}{(zeta -p)(1 -frac{(z-p)}{zeta -p}})$=$int_{C} f(zeta) sum_{n=0}^{infty} (frac{z-p}{zeta -p})^n$ which is valid for $|frac{z-p}{zeta -p}|<1$.



Geometrically, I interpret this as the points closer to $z$ than the boundary of Center image description here



Or, I can choose to do a different power series.



$f(z)=int_{C} frac{f(zeta)}{zeta -z}dzeta$ = manipulating differently, =$int_{C} f(zeta)(frac{1}{zeta -p -(z-p)})$=$int_{C} f(zeta)(frac{1}{-(z -p)(1 -frac{(zeta-p)}{z -p}})$=$-int_{C} f(zeta) sum_{n=0}^{infty} (frac{zeta-p}{z -p})^n$ which is valid for $|frac{zeta-p}{z -p}|<1$.



Which I am then interpreting as the outside region of the circleenter image description here



My question is, as you may have guessed, Where does the Laurent series given by



$f(z)=sum_{n=1}^{infty} a_n(z-p)^n +sum_{n=1}^{infty} frac{b_n}{(z-p)^n}$ converge? I do not understand where the annulus of convergence can be.



Thanks in advance.










share|cite|improve this question









$endgroup$












  • $begingroup$
    a Laurent series converges between the point where it is constructed around (in your case $p$) and up to the closer singularity to the function that it represent. If the function doesn't have any singularity then the Laurent series is just a Taylor series with radius of convergence infinity
    $endgroup$
    – Masacroso
    Nov 27 '18 at 4:17










  • $begingroup$
    @Masacroso Here yes but in general the domain of convergence can be any anulus, eg. $sum_n (1/2)^{|n|} z^n$ converges for $|z| < 1/2, |1/z| < 1/2 = |z| in (1/2,2)$. The (open) annulus where $sum_n a_n z^n$ converges is $|z|in (1/r,R)$ where $R,r$ are the radius of convergence of $sum_{n ge 0} a_n z^n,sum_{n > 0} a_{-n} z^n$. There is a singularity on $|z| = R, |z| =1/r$
    $endgroup$
    – reuns
    Nov 27 '18 at 4:25


















1












$begingroup$


From the Cauchy integral formula, I get



$f(z)=int_{C} frac{f(zeta)}{zeta -z}dzeta$



manipulating, =$int_{C} f(zeta)(frac{1}{zeta -p -(z-p)})$=$int_{C} f(zeta)(frac{1}{(zeta -p)(1 -frac{(z-p)}{zeta -p}})$=$int_{C} f(zeta) sum_{n=0}^{infty} (frac{z-p}{zeta -p})^n$ which is valid for $|frac{z-p}{zeta -p}|<1$.



Geometrically, I interpret this as the points closer to $z$ than the boundary of Center image description here



Or, I can choose to do a different power series.



$f(z)=int_{C} frac{f(zeta)}{zeta -z}dzeta$ = manipulating differently, =$int_{C} f(zeta)(frac{1}{zeta -p -(z-p)})$=$int_{C} f(zeta)(frac{1}{-(z -p)(1 -frac{(zeta-p)}{z -p}})$=$-int_{C} f(zeta) sum_{n=0}^{infty} (frac{zeta-p}{z -p})^n$ which is valid for $|frac{zeta-p}{z -p}|<1$.



Which I am then interpreting as the outside region of the circleenter image description here



My question is, as you may have guessed, Where does the Laurent series given by



$f(z)=sum_{n=1}^{infty} a_n(z-p)^n +sum_{n=1}^{infty} frac{b_n}{(z-p)^n}$ converge? I do not understand where the annulus of convergence can be.



Thanks in advance.










share|cite|improve this question









$endgroup$












  • $begingroup$
    a Laurent series converges between the point where it is constructed around (in your case $p$) and up to the closer singularity to the function that it represent. If the function doesn't have any singularity then the Laurent series is just a Taylor series with radius of convergence infinity
    $endgroup$
    – Masacroso
    Nov 27 '18 at 4:17










  • $begingroup$
    @Masacroso Here yes but in general the domain of convergence can be any anulus, eg. $sum_n (1/2)^{|n|} z^n$ converges for $|z| < 1/2, |1/z| < 1/2 = |z| in (1/2,2)$. The (open) annulus where $sum_n a_n z^n$ converges is $|z|in (1/r,R)$ where $R,r$ are the radius of convergence of $sum_{n ge 0} a_n z^n,sum_{n > 0} a_{-n} z^n$. There is a singularity on $|z| = R, |z| =1/r$
    $endgroup$
    – reuns
    Nov 27 '18 at 4:25
















1












1








1





$begingroup$


From the Cauchy integral formula, I get



$f(z)=int_{C} frac{f(zeta)}{zeta -z}dzeta$



manipulating, =$int_{C} f(zeta)(frac{1}{zeta -p -(z-p)})$=$int_{C} f(zeta)(frac{1}{(zeta -p)(1 -frac{(z-p)}{zeta -p}})$=$int_{C} f(zeta) sum_{n=0}^{infty} (frac{z-p}{zeta -p})^n$ which is valid for $|frac{z-p}{zeta -p}|<1$.



Geometrically, I interpret this as the points closer to $z$ than the boundary of Center image description here



Or, I can choose to do a different power series.



$f(z)=int_{C} frac{f(zeta)}{zeta -z}dzeta$ = manipulating differently, =$int_{C} f(zeta)(frac{1}{zeta -p -(z-p)})$=$int_{C} f(zeta)(frac{1}{-(z -p)(1 -frac{(zeta-p)}{z -p}})$=$-int_{C} f(zeta) sum_{n=0}^{infty} (frac{zeta-p}{z -p})^n$ which is valid for $|frac{zeta-p}{z -p}|<1$.



Which I am then interpreting as the outside region of the circleenter image description here



My question is, as you may have guessed, Where does the Laurent series given by



$f(z)=sum_{n=1}^{infty} a_n(z-p)^n +sum_{n=1}^{infty} frac{b_n}{(z-p)^n}$ converge? I do not understand where the annulus of convergence can be.



Thanks in advance.










share|cite|improve this question









$endgroup$




From the Cauchy integral formula, I get



$f(z)=int_{C} frac{f(zeta)}{zeta -z}dzeta$



manipulating, =$int_{C} f(zeta)(frac{1}{zeta -p -(z-p)})$=$int_{C} f(zeta)(frac{1}{(zeta -p)(1 -frac{(z-p)}{zeta -p}})$=$int_{C} f(zeta) sum_{n=0}^{infty} (frac{z-p}{zeta -p})^n$ which is valid for $|frac{z-p}{zeta -p}|<1$.



Geometrically, I interpret this as the points closer to $z$ than the boundary of Center image description here



Or, I can choose to do a different power series.



$f(z)=int_{C} frac{f(zeta)}{zeta -z}dzeta$ = manipulating differently, =$int_{C} f(zeta)(frac{1}{zeta -p -(z-p)})$=$int_{C} f(zeta)(frac{1}{-(z -p)(1 -frac{(zeta-p)}{z -p}})$=$-int_{C} f(zeta) sum_{n=0}^{infty} (frac{zeta-p}{z -p})^n$ which is valid for $|frac{zeta-p}{z -p}|<1$.



Which I am then interpreting as the outside region of the circleenter image description here



My question is, as you may have guessed, Where does the Laurent series given by



$f(z)=sum_{n=1}^{infty} a_n(z-p)^n +sum_{n=1}^{infty} frac{b_n}{(z-p)^n}$ converge? I do not understand where the annulus of convergence can be.



Thanks in advance.







complex-analysis






share|cite|improve this question













share|cite|improve this question











share|cite|improve this question




share|cite|improve this question










asked Nov 27 '18 at 4:07









JungleshrimpJungleshrimp

19311




19311












  • $begingroup$
    a Laurent series converges between the point where it is constructed around (in your case $p$) and up to the closer singularity to the function that it represent. If the function doesn't have any singularity then the Laurent series is just a Taylor series with radius of convergence infinity
    $endgroup$
    – Masacroso
    Nov 27 '18 at 4:17










  • $begingroup$
    @Masacroso Here yes but in general the domain of convergence can be any anulus, eg. $sum_n (1/2)^{|n|} z^n$ converges for $|z| < 1/2, |1/z| < 1/2 = |z| in (1/2,2)$. The (open) annulus where $sum_n a_n z^n$ converges is $|z|in (1/r,R)$ where $R,r$ are the radius of convergence of $sum_{n ge 0} a_n z^n,sum_{n > 0} a_{-n} z^n$. There is a singularity on $|z| = R, |z| =1/r$
    $endgroup$
    – reuns
    Nov 27 '18 at 4:25




















  • $begingroup$
    a Laurent series converges between the point where it is constructed around (in your case $p$) and up to the closer singularity to the function that it represent. If the function doesn't have any singularity then the Laurent series is just a Taylor series with radius of convergence infinity
    $endgroup$
    – Masacroso
    Nov 27 '18 at 4:17










  • $begingroup$
    @Masacroso Here yes but in general the domain of convergence can be any anulus, eg. $sum_n (1/2)^{|n|} z^n$ converges for $|z| < 1/2, |1/z| < 1/2 = |z| in (1/2,2)$. The (open) annulus where $sum_n a_n z^n$ converges is $|z|in (1/r,R)$ where $R,r$ are the radius of convergence of $sum_{n ge 0} a_n z^n,sum_{n > 0} a_{-n} z^n$. There is a singularity on $|z| = R, |z| =1/r$
    $endgroup$
    – reuns
    Nov 27 '18 at 4:25


















$begingroup$
a Laurent series converges between the point where it is constructed around (in your case $p$) and up to the closer singularity to the function that it represent. If the function doesn't have any singularity then the Laurent series is just a Taylor series with radius of convergence infinity
$endgroup$
– Masacroso
Nov 27 '18 at 4:17




$begingroup$
a Laurent series converges between the point where it is constructed around (in your case $p$) and up to the closer singularity to the function that it represent. If the function doesn't have any singularity then the Laurent series is just a Taylor series with radius of convergence infinity
$endgroup$
– Masacroso
Nov 27 '18 at 4:17












$begingroup$
@Masacroso Here yes but in general the domain of convergence can be any anulus, eg. $sum_n (1/2)^{|n|} z^n$ converges for $|z| < 1/2, |1/z| < 1/2 = |z| in (1/2,2)$. The (open) annulus where $sum_n a_n z^n$ converges is $|z|in (1/r,R)$ where $R,r$ are the radius of convergence of $sum_{n ge 0} a_n z^n,sum_{n > 0} a_{-n} z^n$. There is a singularity on $|z| = R, |z| =1/r$
$endgroup$
– reuns
Nov 27 '18 at 4:25






$begingroup$
@Masacroso Here yes but in general the domain of convergence can be any anulus, eg. $sum_n (1/2)^{|n|} z^n$ converges for $|z| < 1/2, |1/z| < 1/2 = |z| in (1/2,2)$. The (open) annulus where $sum_n a_n z^n$ converges is $|z|in (1/r,R)$ where $R,r$ are the radius of convergence of $sum_{n ge 0} a_n z^n,sum_{n > 0} a_{-n} z^n$. There is a singularity on $|z| = R, |z| =1/r$
$endgroup$
– reuns
Nov 27 '18 at 4:25












0






active

oldest

votes











Your Answer





StackExchange.ifUsing("editor", function () {
return StackExchange.using("mathjaxEditing", function () {
StackExchange.MarkdownEditor.creationCallbacks.add(function (editor, postfix) {
StackExchange.mathjaxEditing.prepareWmdForMathJax(editor, postfix, [["$", "$"], ["\\(","\\)"]]);
});
});
}, "mathjax-editing");

StackExchange.ready(function() {
var channelOptions = {
tags: "".split(" "),
id: "69"
};
initTagRenderer("".split(" "), "".split(" "), channelOptions);

StackExchange.using("externalEditor", function() {
// Have to fire editor after snippets, if snippets enabled
if (StackExchange.settings.snippets.snippetsEnabled) {
StackExchange.using("snippets", function() {
createEditor();
});
}
else {
createEditor();
}
});

function createEditor() {
StackExchange.prepareEditor({
heartbeatType: 'answer',
autoActivateHeartbeat: false,
convertImagesToLinks: true,
noModals: true,
showLowRepImageUploadWarning: true,
reputationToPostImages: 10,
bindNavPrevention: true,
postfix: "",
imageUploader: {
brandingHtml: "Powered by u003ca class="icon-imgur-white" href="https://imgur.com/"u003eu003c/au003e",
contentPolicyHtml: "User contributions licensed under u003ca href="https://creativecommons.org/licenses/by-sa/3.0/"u003ecc by-sa 3.0 with attribution requiredu003c/au003e u003ca href="https://stackoverflow.com/legal/content-policy"u003e(content policy)u003c/au003e",
allowUrls: true
},
noCode: true, onDemand: true,
discardSelector: ".discard-answer"
,immediatelyShowMarkdownHelp:true
});


}
});














draft saved

draft discarded


















StackExchange.ready(
function () {
StackExchange.openid.initPostLogin('.new-post-login', 'https%3a%2f%2fmath.stackexchange.com%2fquestions%2f3015323%2fwhere-does-a-laurent-series-converge%23new-answer', 'question_page');
}
);

Post as a guest















Required, but never shown

























0






active

oldest

votes








0






active

oldest

votes









active

oldest

votes






active

oldest

votes
















draft saved

draft discarded




















































Thanks for contributing an answer to Mathematics Stack Exchange!


  • Please be sure to answer the question. Provide details and share your research!

But avoid



  • Asking for help, clarification, or responding to other answers.

  • Making statements based on opinion; back them up with references or personal experience.


Use MathJax to format equations. MathJax reference.


To learn more, see our tips on writing great answers.




draft saved


draft discarded














StackExchange.ready(
function () {
StackExchange.openid.initPostLogin('.new-post-login', 'https%3a%2f%2fmath.stackexchange.com%2fquestions%2f3015323%2fwhere-does-a-laurent-series-converge%23new-answer', 'question_page');
}
);

Post as a guest















Required, but never shown





















































Required, but never shown














Required, but never shown












Required, but never shown







Required, but never shown

































Required, but never shown














Required, but never shown












Required, but never shown







Required, but never shown







Popular posts from this blog

Biblatex bibliography style without URLs when DOI exists (in Overleaf with Zotero bibliography)

ComboBox Display Member on multiple fields

Is it possible to collect Nectar points via Trainline?